Consumers

This topic has expert replies
User avatar
Master | Next Rank: 500 Posts
Posts: 400
Joined: Sat Mar 10, 2007 4:04 am
Thanked: 1 times
Followed by:1 members

Consumers

by f2001290 » Mon Jun 18, 2007 9:27 am
Consumers planning to buy recreational equipment tend to buy higher quality,
more expensive equipment when the economy is strong than when it is
weak. Hill and Dale is a business that sells high-quality, expensive camping
and hiking equipment in Boravia. Although all the signs are that Boravia’s
economy is now entering a period of sustained strength, the managers
of the business do not expect a substantial increase in sales.
Which of the following, if true, would provide the strongest
justification for the managers’ judgment?
A. A significant proportion of Hill and Dale’s sales are made to
customers who enter the store in order to buy one particular
item but, once there, find other items to buy as well.
B. In Boravia when the economy is strong, those who might
otherwise go camping tend to take vacations overseas.
C. The economic upturn is likely to allow Boravia’s national
parks, where most of the camping and hiking is
done, to receive extra funding to improve their visitor facilities.
D. Advances in materials technology have led to the
development of hiking and camping equipment that
is more comfortable and lightweight than before.
E. Many people in Boravia not only are committed to
preserving the country’s wilderness areas but also
are interested in spending some time in them.

OA after few explanations.

User avatar
Master | Next Rank: 500 Posts
Posts: 277
Joined: Sun Jun 17, 2007 2:51 pm
Location: New York, NY
Thanked: 6 times
Followed by:1 members

by givemeanid » Mon Jun 18, 2007 10:19 am
The assumption is that Boravians will buy the expensive equipment when the economy is strong. However, managers expectation that the sales won't increase is justified by (B). When the economy is strong, Boravians will go overseas and not buy anything.

Newbie | Next Rank: 10 Posts
Posts: 5
Joined: Mon Jun 11, 2007 10:36 am

by cooldude2281 » Mon Jun 18, 2007 10:37 am
B from my side also

User avatar
Master | Next Rank: 500 Posts
Posts: 400
Joined: Sat Mar 10, 2007 4:04 am
Thanked: 1 times
Followed by:1 members

by f2001290 » Mon Jun 18, 2007 10:54 am
This question is from GMAT Sets. OA is C.

Since all of us agree with B, OA is wrong.

Newbie | Next Rank: 10 Posts
Posts: 5
Joined: Mon Jun 11, 2007 10:36 am

by cooldude2281 » Mon Jun 18, 2007 12:30 pm
by improving faclities more ppl will go and more ppl will buy the equipment. that would be entirely opposite of what is asked.

Senior | Next Rank: 100 Posts
Posts: 64
Joined: Mon Nov 19, 2007 9:53 am
Thanked: 2 times

by jan08 » Sat Dec 08, 2007 4:44 pm
I was stuck b/w B & C..here is why

B: states that when the economy is strong more people will go overseas for vacation rather than do camping..this will lower the sales..hence the managers are correct in their assumption...(this was my initial guess)...

C: states that the camping facilities/ parks are being improved..now there may be a case that the facilities are improved with the camping equipments and visitors need not buy the camping equipments thereby reducing the sale...again manages are correct in their assumption...

C looks like a valid OA but then whats wrong with B?

Can somebody explain?

Master | Next Rank: 500 Posts
Posts: 195
Joined: Sun Oct 21, 2007 4:33 am
Thanked: 10 times

by sankruth » Sun Dec 09, 2007 2:13 pm
B sounds right.

C states that facilities in camping sites have been improved. But because they are camping sites, people will still need camping equipment, even when facilities have improved. Also there is no evidence that suggests that people buy expensive equipment because the facilities are bad.

GMAT/MBA Expert

User avatar
GMAT Instructor
Posts: 2228
Joined: Wed Dec 27, 2006 3:28 pm
Location: Montreal, Canada
Thanked: 639 times
Followed by:694 members
GMAT Score:780

by Stacey Koprince » Fri Nov 07, 2008 8:12 am
I received a PM asking me to respond.

First, I did a quick search, and found some sites that list the OA as B and some that list the OA as C. This is one of the reasons I tell my students not to use the Sets. There are lots of typos and errors riddled throughout. Some of the questions are perfectly fine... but you don't know which are which.

Cons. buy high Q, more $$ RE when economy is strong.
H&D sells high Q, $$ RE in Boravia
Boravia econ becoming strong
BUT H&D not expecting better sales
why?

A. that doesn't explain why people would not buy more
B. Boravia is different from the general trend: people don't camp as much when economy is strong. That sounds pretty decent. Let's check the others.
C. hmm. what are "visitor facilities"? Does that mean trails and port-a-potties? Or does that mean log cabins and other things that would replace the recreational equipment people buy for themselves? This is also tempting, but we have to make some assumptions here, so I don't like this one as much as I like B.
D. that doesn't explain why people would not buy more
E. that doesn't explain why people would not buy more; if anything, sounds like people who want to spend time in wilderness areas would buy more when they could afford to do so.

I noticed on another post that the reasoning against B was that people could go camping overseas. But look at the wording of B: it specifically says "people who might otherwise go camping" - that means they're not going camping now, overseas or at home.

I'd pick B.
Please note: I do not use the Private Messaging system! I will not see any PMs that you send to me!!

Stacey Koprince
GMAT Instructor
Director of Online Community
Manhattan GMAT

Contributor to Beat The GMAT!

Learn more about me

Junior | Next Rank: 30 Posts
Posts: 14
Joined: Thu Apr 16, 2009 4:13 pm

by bend_the_rulz » Tue May 12, 2009 8:46 pm
Thanks Stacey.You rock!
I'd picked B and was scratching my head when I saw the answer as C.

Newbie | Next Rank: 10 Posts
Posts: 4
Joined: Fri Apr 09, 2010 1:14 am

by anilregisgmat » Thu Apr 15, 2010 5:42 pm
I do not think B is an answer. As we can see the Manager does not expect a drop in sales instead he expects a a moderate increase in sales (not substantial). If B were to be true then sales should decrease and is not inline with the sales forecast.

Given all of these C is the better answer. Hence OA is correct

Newbie | Next Rank: 10 Posts
Posts: 3
Joined: Fri Jul 19, 2013 7:15 am

by kshitij90 » Fri Jul 19, 2013 7:22 am
Thanks Stacey, and you're right, this is the umpteenth time that I've got a seemingly right answer incorrect and later found out that it actually waS the right one.

Senior | Next Rank: 100 Posts
Posts: 39
Joined: Fri Feb 06, 2009 9:22 pm
Thanked: 1 times

by bharath2787 » Sun Jul 21, 2013 8:50 am
C sounds good. misleading question. initially i thought A or B but, aren't they both act i na similar way !! ??

A and B clearly refute the premises made by the author. Remember te argument clearly says, COnsumers who want to buy recreational equipement tend to buy the high quality stuff. hence , the argument talks about people who want to buy high quality recreational stuff not general outgoing people.

I would still go with C. please someonw explain if this approach is rite.
Bharath

Senior | Next Rank: 100 Posts
Posts: 39
Joined: Fri Feb 06, 2009 9:22 pm
Thanked: 1 times

by bharath2787 » Sun Jul 21, 2013 9:05 am
Further, Also From B, why cant a vacationer buy and keep the equipment at home, come back and work out :C. moreover, the word Might also suggests a different meaning.
Bharath

Senior | Next Rank: 100 Posts
Posts: 39
Joined: Sat Jan 12, 2019 1:08 am

by meenakshimiyer » Sat Jan 12, 2019 9:59 pm
Conclusion: Hill and Dale thinks even though the economy is entering a period of sustained strength, the sale won't increase.
A. This statement has no relevance to the argument.
B. This is the best strengthening statement to Hill and Dale's argument. If the economy strengthens then people will prefer spending vacations overseas to hiking hence declining the sales of hiking equipments. Hence, B is the answer.
C. This could be a weakening statement. If hiking facilities improve then more people will prefer hiking which will eventually increase the sale of hiking products.
D. This statement has no relevance to the argument.
E. This statement holds little or no relevance to the argument.